AMC 12A 2015
by djmathman, Jan 31, 2015, 2:25 AM
is this Tuesday. My aim is a 120+ so that I don't need to pull an unrealistic performance on the AIME to qualify for USAMO.
I'm actually legitimately scared, oops
In the meantime, here's an OMO problem from the last contest that I finally got around to writing a complete solution to last week.
Hopefully rigorous solution
I'm actually legitimately scared, oops
In the meantime, here's an OMO problem from the last contest that I finally got around to writing a complete solution to last week.
OMO Fall 2014.18 wrote:
We select a real number
uniformly and at random from the interval
. Define
![\[ S = \frac{1}{\alpha} \sum_{m=1}^{1000} \sum_{n=m}^{1000} \left\lfloor \frac{m+\alpha}{n} \right\rfloor. \]](//latex.artofproblemsolving.com/0/9/9/099efafef67403350c582ffa561a65e586668764.png)
Let
denote the probability that
. Compute
.


![\[ S = \frac{1}{\alpha} \sum_{m=1}^{1000} \sum_{n=m}^{1000} \left\lfloor \frac{m+\alpha}{n} \right\rfloor. \]](http://latex.artofproblemsolving.com/0/9/9/099efafef67403350c582ffa561a65e586668764.png)
Let



Hopefully rigorous solution
Switch the order of the summation so that
![\[S=\dfrac1\alpha\sum_{n=1}^{1000}\sum_{m=1}^n\left\lfloor\dfrac{m+\alpha}n\right\rfloor=\dfrac1\alpha\sum_{n=1}^{1000}\left[\sum_{m=1}^n\dfrac{m+\alpha}n-\sum_{m=1}^n\left\{\dfrac{m+\alpha}n\right\}\right].\]](//latex.artofproblemsolving.com/7/0/9/709f35be0f002962f9b84b7488677f6301fca5bd.png)
We now turn our attention to the fractional part sum. Let
and
be integers such that
and
for every
. Then
![\[\sum_{m=1}^n\left\{\dfrac{m+\alpha}n\right\}=\sum_{m=1}^n\left\{\dfrac{nq_m+(r_m+\{\alpha\})}n\right\}=\sum_{m=1}^n\left\{\dfrac{r_m+\{\alpha\}}n\right\}.\]](//latex.artofproblemsolving.com/a/b/f/abf288af5d7623848eac0c7e541443ea1589a149.png)
Since the sequence
consists of
consecutive integers, the sequence
must consist of the integers
(due to the fact that the remainders will "wrap around" at some point). Therefore the leftover sum can be easily rewritten as
![\[\sum_{m=1}^n\left\{\dfrac{r_m+\{\alpha\}}{n}\right\}=\sum_{m=0}^{n-1}\dfrac{m+\{\alpha\}}n.\]](//latex.artofproblemsolving.com/8/1/5/815c5af41da9d2ec14c4e72e5a7399ac912e9646.png)
As a result, we can simplify our original summation:
![\begin{align*}\dfrac1\alpha\sum_{n=1}^{1000}\sum_{m=1}^n\left\lfloor\dfrac{m+\alpha}n\right\rfloor&=\dfrac1\alpha\sum_{n=1}^{1000}\left[\sum_{m=1}^n\dfrac{m+\alpha}n-\sum_{m=0}^{n-1}\dfrac{m+\{\alpha\}}n\right]\\&=\dfrac1\alpha\sum_{n=1}^{1000}\dfrac{n+n(\alpha-\{\alpha\})}n\\&=\dfrac{1000(1+\lfloor\alpha\rfloor)}\alpha.\end{align*}](//latex.artofproblemsolving.com/3/a/7/3a7aad97fa8b121d9f4f1c72e2577b65890e4d5e.png)
If this simplified expression is to be greater than
, then
. Let
. Then
, so
. Combining this with
gives
. The length of this interval is
, which is nonnegative for
. Hence
![\[p=\dfrac1{500}\sum_{k=0}^5\dfrac16(5-k)=\dfrac1{500}\left(\dfrac{1+2+3+4+5}6\right)=\dfrac1{200}\]](//latex.artofproblemsolving.com/0/4/9/049acf6f4fb1f49b98610a37118786fbccf48db9.png)
and the requested answer is
.
![\[S=\dfrac1\alpha\sum_{n=1}^{1000}\sum_{m=1}^n\left\lfloor\dfrac{m+\alpha}n\right\rfloor=\dfrac1\alpha\sum_{n=1}^{1000}\left[\sum_{m=1}^n\dfrac{m+\alpha}n-\sum_{m=1}^n\left\{\dfrac{m+\alpha}n\right\}\right].\]](http://latex.artofproblemsolving.com/7/0/9/709f35be0f002962f9b84b7488677f6301fca5bd.png)
We now turn our attention to the fractional part sum. Let





![\[\sum_{m=1}^n\left\{\dfrac{m+\alpha}n\right\}=\sum_{m=1}^n\left\{\dfrac{nq_m+(r_m+\{\alpha\})}n\right\}=\sum_{m=1}^n\left\{\dfrac{r_m+\{\alpha\}}n\right\}.\]](http://latex.artofproblemsolving.com/a/b/f/abf288af5d7623848eac0c7e541443ea1589a149.png)
Since the sequence




![\[\sum_{m=1}^n\left\{\dfrac{r_m+\{\alpha\}}{n}\right\}=\sum_{m=0}^{n-1}\dfrac{m+\{\alpha\}}n.\]](http://latex.artofproblemsolving.com/8/1/5/815c5af41da9d2ec14c4e72e5a7399ac912e9646.png)
As a result, we can simplify our original summation:
![\begin{align*}\dfrac1\alpha\sum_{n=1}^{1000}\sum_{m=1}^n\left\lfloor\dfrac{m+\alpha}n\right\rfloor&=\dfrac1\alpha\sum_{n=1}^{1000}\left[\sum_{m=1}^n\dfrac{m+\alpha}n-\sum_{m=0}^{n-1}\dfrac{m+\{\alpha\}}n\right]\\&=\dfrac1\alpha\sum_{n=1}^{1000}\dfrac{n+n(\alpha-\{\alpha\})}n\\&=\dfrac{1000(1+\lfloor\alpha\rfloor)}\alpha.\end{align*}](http://latex.artofproblemsolving.com/3/a/7/3a7aad97fa8b121d9f4f1c72e2577b65890e4d5e.png)
If this simplified expression is to be greater than









![\[p=\dfrac1{500}\sum_{k=0}^5\dfrac16(5-k)=\dfrac1{500}\left(\dfrac{1+2+3+4+5}6\right)=\dfrac1{200}\]](http://latex.artofproblemsolving.com/0/4/9/049acf6f4fb1f49b98610a37118786fbccf48db9.png)
and the requested answer is

This post has been edited 3 times. Last edited by djmathman, Mar 5, 2015, 3:42 AM